- PowerScore Staff
- Posts: 5972
- Joined: Mar 25, 2011
- Mon Jan 21, 2013 2:19 pm
#88142
Complete Question Explanation
(The complete setup for this game can be found here: lsat/viewtopic.php?f=167&p=88139#p88139)
The correct answer choice is (C).
The question stem specifies that G is the second passenger to exit, and that L and S are the first two stops, not necessarily in that order:
Of course, if L and S are the first two stops, then F and M must be the last two stops, in some order. If M is third or fourth, then from the second rule we can infer that R cannot be first, and must be third or fourth. From the third rule J cannot be first, and so the only passenger who is available to exit first is V. With V exiting first, J and R must exit third and fourth, not necessarily in that order. Thus, without accounting entirely for the second and fourth rules, this is the current setup for this question:
While the actions of the second and fourth rules limit the number of solutions possible under the scenario above, the information thus far is sufficient to answer this question. Because S is the first or second stop, and R must exit third or fourth, answer choice (C) must be true and is correct.
The following hypothetical eliminates answer choices (A) and (D):
The following hypothetical eliminates answer choice (B):
The following hypothetical eliminates answer choice (E):
(The complete setup for this game can be found here: lsat/viewtopic.php?f=167&p=88139#p88139)
The correct answer choice is (C).
The question stem specifies that G is the second passenger to exit, and that L and S are the first two stops, not necessarily in that order:
Of course, if L and S are the first two stops, then F and M must be the last two stops, in some order. If M is third or fourth, then from the second rule we can infer that R cannot be first, and must be third or fourth. From the third rule J cannot be first, and so the only passenger who is available to exit first is V. With V exiting first, J and R must exit third and fourth, not necessarily in that order. Thus, without accounting entirely for the second and fourth rules, this is the current setup for this question:
While the actions of the second and fourth rules limit the number of solutions possible under the scenario above, the information thus far is sufficient to answer this question. Because S is the first or second stop, and R must exit third or fourth, answer choice (C) must be true and is correct.
The following hypothetical eliminates answer choices (A) and (D):
The following hypothetical eliminates answer choice (B):
The following hypothetical eliminates answer choice (E):
You do not have the required permissions to view the files attached to this post.
Dave Killoran
PowerScore Test Preparation
Follow me on X/Twitter at http://twitter.com/DaveKilloran
My LSAT Articles: http://blog.powerscore.com/lsat/author/dave-killoran
PowerScore Podcast: http://www.powerscore.com/lsat/podcast/
PowerScore Test Preparation
Follow me on X/Twitter at http://twitter.com/DaveKilloran
My LSAT Articles: http://blog.powerscore.com/lsat/author/dave-killoran
PowerScore Podcast: http://www.powerscore.com/lsat/podcast/